Sbornik: Mathematics
RUS  ENG    JOURNALS   PEOPLE   ORGANISATIONS   CONFERENCES   SEMINARS   VIDEO LIBRARY   PACKAGE AMSBIB  
General information
Latest issue
Forthcoming papers
Archive
Impact factor
Guidelines for authors
License agreement
Submit a manuscript

Search papers
Search references

RSS
Latest issue
Current issues
Archive issues
What is RSS



Mat. Sb.:
Year:
Volume:
Issue:
Page:
Find






Personal entry:
Login:
Password:
Save password
Enter
Forgotten password?
Register


Sbornik: Mathematics, 2023, Volume 214, Issue 7, Pages 919–933
DOI: https://doi.org/10.4213/sm9815e
(Mi sm9815)
 

Karatsuba's divisor problem and related questions

M. R. Gabdullina, S. V. Konyagina, V. V. Iudelevichb

a Steklov Mathematical Institute of Russian Academy of Sciences, Moscow, Russia
b Faculty of Mechanics and Mathematics, Lomonosov Moscow State University, Moscow, Russia
References:
Abstract: We prove that
$$ \sum_{p \leq x} \frac{1}{\tau(p-1)} \asymp \frac{x}{(\log x)^{3/2}} \quad\text{and}\quad \sum_{n \leq x} \frac{1}{\tau(n^2+1)} \asymp \frac{x}{(\log x)^{1/2}}, $$
where $\tau(n)=\sum_{d\mid n}1$ is the number of divisors of $n$, and the first sum is taken over prime numbers.
Bibliography: 14 titles.
Keywords: divisor function, sums of values of functions, shifted primes and squares.
Funding agency Grant number
Russian Science Foundation 19-11-00001
Ministry of Science and Higher Education of the Russian Federation 075-15-2022-265
Foundation for the Development of Theoretical Physics and Mathematics BASIS
The work of M. R. Gabdullin on the upper bound in Theorem 1.2 was performed at the Steklov Mathematical Institute of Russian Academy of Sciences and supported by the Russian Science Foundation under grant no. 19-11-00001, https://rscf.ru/en/project/19-11-00001/. The work of S. V. Konyagin was performed at the Steklov International Mathematical Center and supported by the Ministry of Science and Higher Education of the Russian Federation (agreement no. 075-15-2022-265). The work of V. V. Iudelevich was supported by the Theoretical Physics and Mathematics Advancement Foundation “BASIS”.
Received: 25.07.2022 and 31.03.2023
Russian version:
Matematicheskii Sbornik, 2023, Volume 214, Number 7, Pages 27–41
DOI: https://doi.org/10.4213/sm9815
Bibliographic databases:
Document Type: Article
MSC: 11N35, 11N45
Language: English
Original paper language: Russian

§ 1. Introduction

In 2004 Karatsuba posed the following problem at his seminar “Analytic number theory and applications”: find the asymptotic behaviour of the sum

$$ \begin{equation*} \Phi (x)=\sum_{p \leqslant x} \frac{1}{\tau(p-1)} \end{equation*} \notag $$
as $x\to\infty$, where $\tau(n)=\sum_{d\mid n} 1$ is the divisor function and the sum is taken over the prime numbers not exceeding $x$. This is a natural ‘hybrid’ of the following two classical problems in analytic number theory.

The first (so-called Titchmarsh divisor problem) asks for the asymptotic behaviour of the sum

$$ \begin{equation*} D(x)=\sum_{p\leqslant x}\tau(p-1). \end{equation*} \notag $$
It is well known (see [1]–[4]) that
$$ \begin{equation*} D(x) \sim \frac{\zeta(2)\zeta(3)}{\zeta(6)}x, \qquad x\to\infty, \end{equation*} \notag $$
where $\zeta(s)$ denotes the Riemann zeta function. The other problem is to find the asymptotics for the sum
$$ \begin{equation*} T(x)=\sum_{n\leqslant x}\frac{1}{\tau(n)}. \end{equation*} \notag $$
This problem was solved by Ramanujan [5], who showed that
$$ \begin{equation} T(x)=c_0\frac{x}{\sqrt{\log x}}\biggl( 1+O\biggl(\frac{1}{\sqrt{\log x}} \biggr)\biggr) , \end{equation} \tag{1.1} $$
where
$$ \begin{equation*} c_0=\frac{1}{\sqrt{\pi}} \prod_{p} \sqrt{p^2 - p} \log \frac{p}{p-1}= 0.5486\dots\,. \end{equation*} \notag $$

The sum $\Phi(x)$ was studied previously. In the recent work [6] it was shown that

$$ \begin{equation*} \Phi(x)\leqslant 4K\frac{x}{(\log x)^{3/2}}+O\biggl( \frac{x\log\log x}{(\log x)^{5/2}}\biggr), \end{equation*} \notag $$
where
$$ \begin{equation*} K=\frac{1}{\sqrt{\pi}}\prod_{p}\sqrt{\frac{p}{p-1}}\biggl(p\log\frac{p}{p-1} -\frac{1}{p-1}\biggr)=0.2532\dots\,. \end{equation*} \notag $$
Note that the bound $\Phi(x)\ll \frac{x}{(\log x)^{3/2}}$ follows from Corollary 1.2 in [7] and can also be derived by arguing similarly to the proof of the upper bound in Theorem 1.2 below. We make the conjecture that
$$ \begin{equation*} \Phi(x) \sim K\frac{x}{(\log x)^{3/2}}, \qquad x\to\infty, \end{equation*} \notag $$
but this is probably hard to show.

In this paper we prove that the aforementioned upper bound for $\Phi(x)$ is sharp up to a constant.

Theorem 1.1. We have

$$ \begin{equation*} \Phi (x) \gg \frac{x}{(\log x)^{3/2}}. \end{equation*} \notag $$

Thus, we find the correct order of magnitude of the sum $\Phi(x)$:

$$ \begin{equation*} \Phi(x)\asymp \frac{x}{(\log x)^{3/2}}, \end{equation*} \notag $$
which answers Karatsuba’s question in the first approximation.

In addition to $\Phi(x)$, we consider the sum

$$ \begin{equation*} F(x)=\sum_{n \leqslant x} \frac{1}{\tau(n^2+1)} \end{equation*} \notag $$
and establish the following.

Theorem 1.2. We have

$$ \begin{equation*} F(x)\asymp \frac{x}{(\log x)^{1/2}}. \end{equation*} \notag $$

Now we discuss the main ideas of the proofs. In the sum $\Phi(x)$, for each prime number ${p\leqslant x}$ we write $p-1$ in the form $ab$, where $a$ consists of the prime factors not exceeding $z$, and $b$ has only prime factors greater than $z$; here $z=x^\varepsilon$ for some small fixed $\varepsilon>0$. Now $\Phi(x)$ can be rewritten as

$$ \begin{equation} \Phi(x)=\sum_{\substack{a \leqslant x \\ p\mid a \ \Rightarrow\ p \leqslant z}} \frac{1}{\tau(a)} \sum_{\substack{b \leqslant (x-1)/a \\ p\mid b \ \Rightarrow\ p>z \\ ab +1\text{ is prime}}} \frac{1}{\tau(b)}, \end{equation} \tag{1.2} $$
and, since $\tau(b)=O_\varepsilon(1)$, we have
$$ \begin{equation*} \Phi(x)\gg_\varepsilon \sum_{a\leqslant x^\varepsilon} \frac{1}{\tau(a)} \sum_{\substack{b \leqslant (x-1)/a \\ p\mid b \ \Rightarrow\ p>x^\varepsilon \\ ab +1\text{ is prime}}} 1. \end{equation*} \notag $$
Using the Brun-Hooley sieve one can estimate the inner sum from below by a quantity of order
$$ \begin{equation*} \frac{x}{a(\log x)^2}-R(x;a), \end{equation*} \notag $$
where the contribution from $R(x;a)$ to the outer sum is negligible. Combining these estimates, we see that
$$ \begin{equation*} \Phi(x) \gg_\varepsilon \frac{x}{(\log x)^2}\sum_{a\leqslant x^\varepsilon} \frac{1}{a\tau(a)}\gg_\varepsilon \frac{x}{(\log x)^{3/2}}, \end{equation*} \notag $$
which gives us the required lower bound. Note that this argument does not yield any upper bound for $\Phi(x)$, since the estimation of the contribution of the numbers $a>x^{1-\varepsilon}$ to (1.2) is actually equivalent to the initial problem.

A lower bound for the sum $F(x)$ is obtained in a similar way. The upper bound for $F(x)$ follows from Theorem 1 in [8]. We also note that the same upper bound can be derived from the inequality $\tau(n)\geqslant 2^{\omega(n)}$ (here $\omega(n)$ denotes the number of distinct prime divisors of $n$) and a bound for the number of integers $n\leqslant x$ with a fixed value of $\omega(n^2+1)$. For completeness, we provide this argument.

We observe that the methods used in this work can also be applied to other functions similar to $\tau(n)$. For instance, let $\tau_k(n)$ be the generalized divisor function, $\tau_k(n)=\sum_{n=d_1 d_2\dotsb d_k}1$; then one can show that

$$ \begin{equation*} \sum_{p\leqslant x}\frac{1}{\tau_k(p-1)}\asymp_k x (\log x)^{1/k-2}. \end{equation*} \notag $$

§ 2. Notation

By $\varphi(n) = \#\{k \leqslant n\colon (k,n) = 1\}$ we denote the Euler totient function, and we use $P^{+}(n)$ and $P^{-}(n)$ for the least and greatest prime divisors of the integer $n>1$, respectively; by convention, $P^+(1)=0$ and $P^-(1)=\infty$. We denote by $\pi(x)$ the number of primes not exceeding $x$, and $\pi(x;q,a)$ is the number of primes not exceeding $x$ and belonging to the arithmetic progression $a\ (\operatorname{mod}q)$; we also set $R(x;q,a)=\pi(x;q,a)-{\pi(x)}/{\varphi(q)}$. The notation $f(x)\ll g(x)$, as well as $f(x) = O(g(x))$, means that $|f(x)|\leqslant C g(x)$ for some absolute constant $C>0$ and all possible values of $x$. We write $f(x)\asymp g(x)$ if $f(x)\ll g(x)\ll f(x)$, and we write $f(x)\ll_k g(x)$ if we want to stress that the implied constant depends on $k$.

Now we recall some notation from the sieve methods. Let $\mathcal{A}$ be a finite set of positive integers and $\mathcal{P}$ be a finite set of prime numbers. We define $P = \prod_{p \in \mathcal{P}} p$ and $S(\mathcal{A},\mathcal{P}) = \#\{a \in \mathcal{A}\colon (a,P)=1 \}$. Also let $\mathcal{A}_d=\#\{a \in \mathcal{A}\colon a \equiv 0 \ (\operatorname{mod}d)\}$. We assume that, for all $d \mid P$,

$$ \begin{equation} \mathcal{A}_d=Xg(d)+r_d, \end{equation} \tag{2.1} $$
where $g(d)$ is a multiplicative function such that $0<g(p)<1$ for $p \in \mathcal{P}$ and $g(p)=0$ for $p \notin \mathcal{P}$.

Further, let the set $\mathcal{P}$ be divided into disjoint subsets $\mathcal{P}_1, \mathcal{P}_2, \dots, \mathcal{P}_t$, and let $P_j= \prod_{p \in \mathcal{P}_j} p$. Finally, let $\{k_j\}_{r=1}^t$ be a sequence of even integers. Then we set

$$ \begin{equation*} \begin{gathered} \, V_j=\prod _{p \in \mathcal{P}_r} (1-g(p)), \qquad L_j=\log V_j^{-1}, \qquad E=\sum _{j=1}^{t} \frac{e^{L_j} (L_j)^{k_j+1}}{(k_j+1)!}, \\ R=\sum _{\substack{d_j\mid P_j\\ \omega(d_j) \leqslant k_j}} |r_{d_1 \dotsb d_t}| \quad\text{and}\quad R'=\sum _{l=1}^t \sum _{\substack{d_j\mid P_j \\ \omega(d_j) \leqslant k_j,\, j \ne l \\ \omega(d_l)=k_l+1}} |r_{d_1 \dotsb d_t}|. \end{gathered} \end{equation*} \notag $$

§ 3. Auxiliary results

We need the following version of the Brun-Hooley sieve, which is due to Ford and Halberstam [9].

Theorem 3.1. Let assumption (2.1) be met. Then

$$ \begin{equation*} S(\mathcal{A}, \mathcal{P}) \leqslant X \prod_{p \in \mathcal{P}} (1-g(p))e^{E} + R \end{equation*} \notag $$
and
$$ \begin{equation*} S(\mathcal{A}, \mathcal{P}) \geqslant X \prod_{p \in \mathcal{P}} (1-g(p))(1-E) - R - R'. \end{equation*} \notag $$

For a proof, see [9].

Lemma 3.2. Let $a \leqslant x^{1/40}$ be an even positive integer, and let

$$ \begin{equation*} F_a(x)=\#\biggl\{ n \leqslant \frac{x-1}a\colon an+1\textit{ is a prime number and } P^{-}(n)>x^{{1}/{40}} \biggr\}. \end{equation*} \notag $$
Then for $x\geqslant x_0$,
$$ \begin{equation*} F_a(x) \geqslant \frac{c_1\pi(x)}{\varphi(a)\log x}-R_1, \end{equation*} \notag $$
where $c_1>0$ is an absolute constant and
$$ \begin{equation*} 0\leqslant R_1\leqslant \sum_{d\leqslant x^{13/40}}|R(x;ad,1)|. \end{equation*} \notag $$

Remark 3.3. Similar upper bounds without the error term $R_1$ can be obtained by sifting the set

$$ \begin{equation*} \biggl\{(an+1)(n+P)\colon n\leqslant\frac{x-1}a\biggr\}, \quad\text{where } P=\prod_{p\leqslant z}p. \end{equation*} \notag $$

Proof of Lemma 3.2. We apply Theorem 3.1 to the sets
$$ \begin{equation*} \mathcal{A}=\biggl\{n \leqslant \frac{x-1}a\colon an + 1\text{ is prime}\biggr\} \end{equation*} \notag $$
and $\mathcal{P} = \{p \leqslant z\}$, where $z=x^{1/40}$ (the other parameters will be chosen later). Then $F_a(x)=S(\mathcal{A}, \mathcal{P})$, and for each $d\mid P=\prod_{p\leqslant z}p$ we have
$$ \begin{equation*} \mathcal{A}_d=\#\biggl\{k\leqslant \frac{x-1}{ad}\colon adk+1\text{ is prime}\biggr\} =\pi(x; ad,1)=\frac{\pi(x)}{\varphi(ad)}+R(x; ad,1). \end{equation*} \notag $$
Now we write a number $d\mid P$ in the form $d=d_1d_2$, where $(d_1,a)=1$ and $d_2$ consists only of primes dividing $a$. Then, since $\varphi(ad_2)=ad_2\prod_{p\mid a}(1-1/p)=d_2\varphi(a)$, we find that
$$ \begin{equation*} \frac{1}{\varphi(ad)}=\frac{1}{\varphi(d_1)\varphi(ad_2)} =\frac{1}{\varphi(d_1)d_2\varphi(a)}, \end{equation*} \notag $$
hence (2.1) holds for $X= {\pi(x)}/{\varphi(a)}$, the multiplicative function $g$ defined by
$$ \begin{equation*} g(p)=\begin{cases} \dfrac{1}{p-1}&\text{if}\ (p,a)=1, \\ \dfrac{1}{p}&\text{otherwise}, \end{cases} \end{equation*} \notag $$
and
$$ \begin{equation*} r_d=R(x; ad, 1). \end{equation*} \notag $$
In addition, we have
$$ \begin{equation*} \prod_{p \in \mathcal{P}}(1-g(p))\asymp \frac{1}{\log x}, \end{equation*} \notag $$
with an absolute implied constant.

Now we choose a partition of $\mathcal{P}$ and define the numbers $\{k_j\}_{j=1}^t$. Set $z_j= z^{2^{1-j}}$ and

$$ \begin{equation*} \mathcal{P}_j=\mathcal{P} \cap (z_{j+1}, z_j]; \end{equation*} \notag $$
then $t$ is the unique positive integer such that $z_{t+1} < 2 \leqslant z_t$. Let $k_j = b + 2(j-1)$, where $b \geqslant 2$ is even and will be chosen later. We also set
$$ \begin{equation*} C=\prod_{p>2}\biggl(1+\frac{1}{p^2-2p}\biggr)\leqslant 1.52. \end{equation*} \notag $$
By [10], Theorem 7 (see (3.26) and (3.27) in that paper), for any $x>1$,
$$ \begin{equation} \frac{e^{-\gamma}}{\log x}\biggl(1-\frac{1}{\log^2x}\biggr) < \prod_{p\leqslant x}\biggl(1-\frac1p\biggr) < \frac{e^{-\gamma}}{\log x}\biggl(1+\frac{1}{2\log^2x}\biggr), \end{equation} \tag{3.1} $$
where $\gamma$ is the Euler constant. Hence, for any $z\geqslant \sqrt2$ we have
$$ \begin{equation} \prod_{z<p\leqslant z^2}\biggl(1-\frac1p\biggr)^{-1}\leqslant 3; \end{equation} \tag{3.2} $$
indeed, this follows from (3.1) for $z\geqslant 4$ and can be checked manually for ${\sqrt{2}\,{\leqslant}\,z\,{<}\,4}$. Thus,
$$ \begin{equation*} V_j^{-1}=\prod_{z_{j+1} < p \leqslant z_j} (1 - g(p))^{-1} \leqslant C\prod_{z_{j+1}<p\leqslant z_j}\biggl(1-\frac1p\biggr)^{-1} \leqslant 3C\leqslant 5. \end{equation*} \notag $$
Hence $L_j = \log V_j^{-1}\leqslant L=\log 5$ and
$$ \begin{equation*} E=\sum_{j=1}^t \frac{e^{L_j} L_j^{k_j+1}}{(k_j+1)!} \leqslant e^L\sum_{j=1}^t\frac{L^{b+2j-1}}{(b+2j-1)!}. \end{equation*} \notag $$

Now we estimate $R+R'$. If an integer $d$ corresponds to a summand in $R$, then $d=d_1 \dotsb d_t$, where $d_j \mid {P}_j$ and $\omega(d_j) \leqslant k_j=b+2(j-1)$. Therefore,

$$ \begin{equation*} d \leqslant z_1^{k_1} \dotsb z_t^{k_t} \leqslant z^{b+(b+2)/2+(b+4)/4+\dotsb}=z^{2b+4}. \end{equation*} \notag $$
If a number $d$ corresponds to a summand in $R'$, then we find similarly that ${d\leqslant z^{2b+4} z = z^{2b+5}}$. Since the numbers $d$ corresponding to the sums $R$ and $R'$ are distinct and do not exceed $z^{2b+5}$, we see that
$$ \begin{equation*} R+R' \leqslant \sum_{d \leqslant z^{2b+5}} |r_d|. \end{equation*} \notag $$
Now we take $b=4$. Then $2b+5=13$,
$$ \begin{equation*} E\leqslant 5\sum_{j=1}^{\infty}\frac{(\log 5)^{2j+3}}{(2j+3)!} \leqslant 0.48 \end{equation*} \notag $$
and
$$ \begin{equation*} R+R' \leqslant \sum_{d \leqslant x^{13/40}} |R(x;ad,1)|. \end{equation*} \notag $$
The claim follows.

We set

$$ \begin{equation*} \mathcal{M}=\bigl\{ a \geqslant 1\colon a \mid n^2 + 1 \text{ for some } n \geqslant 1 \bigr\}. \end{equation*} \notag $$
It is well known that $a \in \mathcal{M}$ if and only if $4 \nmid a$ and $p \nmid a$ for any $p \equiv 3\ (\operatorname{mod}4)$.

Lemma 3.4. Let $2\leqslant a, z\leqslant x^{1/30}$, $a \in \mathcal{M}$ and $P^{+}(a)\leqslant z$. Let

$$ \begin{equation*} W_a(x,z)=\# \biggl\{ n \leqslant x\colon a\mid (n^2 + 1) \textit{ and } P^-\biggl(\frac{n^2+1}{a}\biggr) > z \biggr\}. \end{equation*} \notag $$
Then
$$ \begin{equation*} W_a(x,z) \asymp \frac{2^{\omega(a)}}{\varphi(a)}\,\frac{x}{\log z} . \end{equation*} \notag $$

Proof. Setting $\mathcal{A}=\{k\colon ak=n^2+1 \text{ for some } n\leqslant x\}$ and
$$ \begin{equation*} \mathcal{P}= \begin{cases} \{p \leqslant z\colon p\equiv 1\ (\operatorname{mod}4)\} &\text{if $a$ is even}, \\ \{2\}\cup \{p \leqslant z\colon p\equiv 1\ (\operatorname{mod}4)\} &\text{if $a$ is odd}. \end{cases} \end{equation*} \notag $$
We have $W_a(x,z) = S(\mathcal{A}, \mathcal{P})$. As before, we write each $d$ dividing $ P=\prod_{p\in\mathcal{P}}p$ as $d=d_1d_2$, where $(d_1,a)=1$ and $d_2$ consists only of primes dividing $a$. Suppose that one of the integers $d$ and $a$ is even. Then by the Chinese remainder theorem and the fact that the congruence $x^2+1\equiv0\ (\operatorname{mod}p)$ has two solutions for $p\equiv 1\ (\operatorname{mod}4)$ and one solution for $p=2$, we obtain
$$ \begin{equation*} \begin{aligned} \, \mathcal{A}_d &=\#\{n\leqslant x\colon n^2+1 \equiv 0 \ (\operatorname{mod}ad) \} \\ &= \frac{x2^{\omega(ad)-1}}{ad}+O(2^{\omega(ad)}) =\frac{x2^{\omega(a)+\omega(d_1)-1}}{ad_1d_2}+O(2^{\omega(ad)}). \end{aligned} \end{equation*} \notag $$
If both $a$ and $d$ are odd, then a similar argument shows that
$$ \begin{equation*} \mathcal{A}_d= \frac{x2^{\omega(a)+\omega(d_1)}}{ad_1d_2}+O(2^{\omega(ad)}). \end{equation*} \notag $$
In both cases we have
$$ \begin{equation*} \mathcal{A}_d=\frac{x2^{\omega(a)+\omega(d_1)-\mathbb I(2\mid ad)}}{ad_1d_2}+O(2^{\omega(ad)}) =\frac{x2^{\omega(a)-\mathbb I(2\mid a)}}{a}\frac{2^{\omega(d_1)-\mathbb I(2\mid d_1)}}{d_1d_2}+O(\tau(ad)), \end{equation*} \notag $$
where $\mathbb I(2\mid l)$ is equal to one if $l$ is even and to zero otherwise. Thus we see that condition (2.1) holds for $X= (x2^{\omega(a)-\mathbb I(2\mid a)})/a$, the multiplicative function $g$ defined on the prime numbers in $\mathcal{P}$ by
$$ \begin{equation*} g(p)=\begin{cases} \dfrac{2}{p} &\text{if } p\nmid a, \\ \dfrac{1}{p} &\text{if } p\mid a \end{cases} \end{equation*} \notag $$
(and also $g(2)=1/2$ in the case of odd $a$), and
$$ \begin{equation*} r_d=O(\tau(ad)). \end{equation*} \notag $$
It is well known that
$$ \begin{equation} \prod _{\substack{p\leqslant x \\ p\equiv 1\, (\operatorname{mod}4)}}\biggl(1-\frac{1}{p} \biggr)\asymp \frac{1}{\sqrt{\log x}} \end{equation} \tag{3.3} $$
(see [11]). Thus, in both cases in the definition of $\mathcal{P}$ we have
$$ \begin{equation*} \prod_{p \in \mathcal{P}}(1-g(p))\asymp \prod_{p\in \mathcal{P}, \, p>2}\biggl(1-\frac2p\biggr) \prod_{p\mid a,\,p>2}\frac{1-1/p}{1-2/p} \asymp \frac{a}{\varphi(a)\log z} \end{equation*} \notag $$
for an absolute implied constant.

Now we choose a partition of $\mathcal{P}$ and define the numbers $\{k_j\}_{j=1}^t$. Again, we set $z_j= z^{2^{1-j }}$ and

$$ \begin{equation*} \mathcal{P}_j=\mathcal{P} \cap (z_{j+1}, z_j], \end{equation*} \notag $$
where $t$ is the unique positive integer such that $z_{t+1} < 2 \leqslant z_t$ and $k_j = b + 2(j-1)$ for some even $b \geqslant 2$. Now (3.2) implies that
$$ \begin{equation*} \begin{aligned} \, V_j^{-1} &=\prod_{z_{j+1} < p \leqslant z_j} (1 - g(p))^{-1} \leqslant 2\prod_{z_{j+1}<p\leqslant z_j, \, p\neq2}\biggl(1-\frac2p\biggr)^{-1} \\ &=2\prod_{z_{j+1}<p\leqslant z_j, \, p\neq2}\biggl(1-\frac1p\biggr)^{-2}\frac{(1-1/p)^2}{1-2/p} \leqslant 18C\leqslant 28. \end{aligned} \end{equation*} \notag $$
Therefore, $L_j = \log V_j^{-1}\leqslant L=\log 28$ and
$$ \begin{equation*} E=\sum_{j=1}^t \frac{e^{L_j} L_j^{k_j+1}}{(k_j+1)!} \leqslant e^L\sum_{j=1}^t\frac{L^{b+2j-1}}{(b+2j-1)!}. \end{equation*} \notag $$
Now we take $b=10$; then $E\leqslant 0.43$ and $2b+5=25$. As in the proof of Lemma 3.2, we obtain
$$ \begin{equation*} R+R' \leqslant \sum_{d \leqslant z^{2b+5}} |r_d|\ll \tau(a)\sum_{d\leqslant z^{25}}\tau(d)\ll x^{1/30}z^{25} \ll x^{26/30}, \end{equation*} \notag $$
and the main term is at least
$$ \begin{equation*} X\prod_{p\in \mathcal{P}}(1-g(p)) \asymp\frac{x2^{\omega(a)}}{\varphi(a)\log z} \gg\frac{x^{1-1/30}}{\log x}. \end{equation*} \notag $$
An application of Theorem 3.1 completes the proof.

The next lemma is a special case of a result due to Tenenbaum [12]. For completeness, we provide the proof.

Lemma 3.5. Let $R>0$ be fixed. Then there exist positive constants $A=A(R)$ and $B=B(R)$ such that, for all $1\leqslant k \leqslant R\log\log x$,

$$ \begin{equation*} \#\bigl\{n\leqslant x\colon \omega(n^2+1)=k\bigr\} \leqslant \frac{Ax(\log\log x+B)^{k-1}}{(k-1)!\, \log x}. \end{equation*} \notag $$

Proof. For $m\geqslant 2$ and $y\geqslant2$ we define the ‘$y$-smooth part’ of $m$ by
$$ \begin{equation*} d(m,y)=\max\bigl\{d\mid m\colon P^+(d)\leqslant y\bigr \}. \end{equation*} \notag $$
For each $n\leqslant x$ the number $n^2+1$ can be written as the product $ab$, where
$$ \begin{equation*} a=a(n)=\max\bigl\{d(n^2+1,y)\colon d(n^2+1,y)\leqslant x^{1/30} \bigr\} \end{equation*} \notag $$
and $b=b(n)=(n^2+1)/a$; note that $(a,b)=1$. Now we set
$$ \begin{equation*} \begin{gathered} \, A_1 =\bigl\{ n\leqslant x\colon a\leqslant x^{1/60},\, P^-(b)>x^{1/60},\, b>1\bigr\}, \\ A_2 =\bigl\{ n\leqslant x\colon a\leqslant x^{1/60},\, P^-(b)\leqslant x^{1/60},\, b>1\bigr\} \end{gathered} \end{equation*} \notag $$
and
$$ \begin{equation*} A_3 =\bigl\{ n\leqslant x\colon x^{1/60}< a \leqslant x^{1/30},\, b>1 \bigr\}. \end{equation*} \notag $$
Since the number of integers $n\leqslant x$ for which $b(n)=1$ does not exceed $x^{1/60}$, we conclude that
$$ \begin{equation} \#\bigl\{n\leqslant x\colon \omega(n^2+1)=k\bigr\}=N_1+N_2+N_3+O(x^{1/60}), \end{equation} \tag{3.4} $$
where
$$ \begin{equation*} N_i=\#\bigl\{n\in A_i\colon \omega(n^2+1)=k \bigr\}, \qquad i=1, 2, 3. \end{equation*} \notag $$

First we estimate $N_1$. In this case $1\leqslant \omega(b)\leqslant 120$; by Lemma 3.4 we have

$$ \begin{equation} \begin{aligned} \, \notag N_1 &\leqslant \sum_{l=k-120}^{k-1}\sum_{\substack{a\leqslant x^{1/60}, \, a\in \mathcal{M} \\ \omega(a)=l }} W_a(x, x^{1/60}) \ll \sum_{l=k-120}^{k-1}\sum_{\substack{a\leqslant x^{1/60}, \, a\in \mathcal{M} \\ \omega(a)=l }}\frac{x2^{\omega(a)}}{\varphi(a)\log x} \\ &\leqslant \frac{x}{\log x}\sum_{l=k-120}^{k-1}2^l\sum_{\substack{a\leqslant x^{1/60}, \, a\in \mathcal{M} \\ \omega(a)=l }}\frac{1}{\varphi(a)}. \end{aligned} \end{equation} \tag{3.5} $$
Taking the logarithm of both sides of (3.3) we obtain
$$ \begin{equation*} \sum _{\substack{p\leqslant x \\ p\equiv 1\, (\operatorname{mod}4)}}\frac{1}{p}= \frac{1}{2}\log\log x+O(1). \end{equation*} \notag $$
Hence for each $l\geqslant 0$ the inner sum in (3.5) is at most
$$ \begin{equation*} \frac{1}{l!}\biggl(\,\sum_{p\leqslant x^{1/60},\, p\neq 3\, (\operatorname{mod}4)}\frac{1}{\varphi(p)}+\frac{1}{\varphi(p^2)}+\dotsb \biggr)^l \leqslant \frac{1}{l!}\biggl(\frac{1}{2}\log\log x+B_1\biggr)^l \end{equation*} \notag $$
for some absolute positive constant $B_1$. Thus,
$$ \begin{equation} N_1\ll \frac{x(\log\log x+B_1)^{k-1}}{(k-1)!\,\log x}\bigl(1+R+\dots+R^{120} \bigr). \end{equation} \tag{3.6} $$

Now we estimate $N_2$. Let $p=P^-(b)$, and let $r$ be the largest integer such that $p^r\mid b$. Then by the definition of $a$, we have $ap^r>x^{1/30}$. Thus, for any $n\in A_2$ we have $p^r> x^{1/60}$, and since $p\leqslant x^{1/60}$, we see that $r\geqslant 2$. Let $\nu = \min\{{u\geqslant 1}$: ${p^u>x^{1/60}}\}$. Then $2\leqslant \nu\leqslant r$ and $p^{\nu-1}\leqslant x^{1/60}$. Hence ${p^\nu\leqslant x^{1/60}p\leqslant x^{1/30}}$. We conclude that for each $n\in A_2$ the integer $n^2+1$ is divisible by $p^\nu$, where $p$ is a prime number such that $x^{1/60}<p^\nu\leqslant x^{1/30}$ and $\nu\geqslant 2$. Therefore,

$$ \begin{equation*} N_2\leqslant\sum_{x^{1/60}<p^\nu\leqslant x^{1/30},\,\nu\geqslant 2}\biggl( \frac{2x}{p^\nu}+O(1)\biggr)\ll \sum_{x^{1/60}<p^\nu \leqslant x^{1/30},\,\nu\geqslant 2}\frac{x}{p^\nu}. \end{equation*} \notag $$
Note that $p^{\nu} \geqslant \max\{p^2, x^{1/60}\}$, and therefore $p^{\nu}\geqslant px^{1/120}$. It follows that
$$ \begin{equation} N_2\ll \sum_{p\leqslant x^{1/30}}\frac{x}{px^{1/120}} \ll x^{1-1/120}\log\log x. \end{equation} \tag{3.7} $$

Finally, consider $N_3$. Setting $q=P^+(a)$, we have $P^-(b)\geqslant q+1$ and

$$ \begin{equation} \omega(b)\leqslant \frac{\log (x^2+1)}{\log (q+1)}\leqslant \frac{2\log x}{\log q}=:\eta. \end{equation} \tag{3.8} $$
Lemma 3.4 implies that
$$ \begin{equation} \begin{aligned} \, N_3 &\leqslant \sum_{\substack{q\leqslant x^{1/30} \\ q\not{\equiv} 3\, (\operatorname{mod}4)}}\sum_{k-\eta\leqslant l\leqslant k-1}\sum_{\substack{x^{1/60}<a\leqslant x^{1/30} \\ P^+(a)=q, \, \omega(a)=l}}W_a(x,q) \nonumber \\ &\ll x\sum_{\substack{q\leqslant x^{1/30} \\ q\not{\equiv} 3\, (\operatorname{mod}4)}}\frac{1}{\log q}\sum_{k-\eta\leqslant l\leqslant k-1}2^l\sum_{\substack{x^{1/60}<a\leqslant x^{1/30} \\ P^+(a)=q, \, \omega(a)=l}}\frac{1}{\varphi(a)}. \end{aligned} \end{equation} \tag{3.9} $$
Let
$$ \begin{equation} \delta=\frac{C}{\log q}, \quad\text{where } C=120\log(R+2)+60. \end{equation} \tag{3.10} $$
Let us denote by $N_3^{(1)}$ and $N_3^{(2)}$ the contributions from $q\leqslant e^{2C}$ and $q>e^{2C}$, respectively. Since
$$ \begin{equation*} \varphi(a)\gg\frac{a}{\log\log a}\gg\frac{x^{1/60}}{\log\log x}, \end{equation*} \notag $$
we have
$$ \begin{equation} N_3^{(1)}\ll x^{59/60}\log\log x\sum_{q\leqslant e^{2C}}\sum_{1\leqslant l\leqslant \pi(q)}2^l\sum_{\substack{a\leqslant x^{1/30} \\ P^+(a)=q}}1 \ll x^{59/60}(\log x)^{c_R}, \end{equation} \tag{3.11} $$
where $c_R>0$ depends only on $R$. Now we turn to $N_3^{(2)}$. For fixed $q$ and $l$ we see that
$$ \begin{equation*} \begin{aligned} \, S_{q,l} &:=\sum_{\substack{x^{1/60}<a\leqslant x^{1/30} \\ P^+(a)=q, \, \omega(a)=l}}\frac{1}{\varphi(a)} \leqslant \sum_{\substack{a\leqslant x^{1/30} \\ P^+(a)=q, \, \omega(a)=l}}\biggl(\frac{a}{x^{1/60}}\biggr)^{\delta}\frac{1}{\varphi(a)} \\ &\leqslant x^{-\delta/60}\frac{1}{(l-1)!}\biggl(\sum_{\substack{p<q \\ p\not{\equiv} 3\, (\operatorname{mod}4)}}\frac{p^{\delta}}{\varphi(p)} +\frac{p^{2\delta}}{\varphi(p^2)}+\dotsb \biggr)^{l-1}\biggl(\frac{q^{\delta}}{\varphi(q)} +\frac{q^{2\delta}}{\varphi(q^2)}+\dotsb\biggr). \end{aligned} \end{equation*} \notag $$
Note that
$$ \begin{equation*} \frac{q^{\delta}}{\varphi(q)}+\frac{q^{2\delta}}{\varphi(q^2)}+\dotsb \ll_R \frac1q \quad\text{for } q>e^{2C}, \end{equation*} \notag $$
and the sum over the primes $p$ is (since $e^u\leqslant 1+O_K(u)$ for $0\leqslant u\leqslant K$) equal to
$$ \begin{equation*} \begin{aligned} \, &\sum_{\substack{p<q \\ p\not{\equiv} 3\, (\operatorname{mod}4)}}\frac{p^\delta}{p}+O_R(1) \\ &\qquad=\sum_{\substack{p<q \\ p\not{\equiv} 3\, (\operatorname{mod}4)}}\frac{1}{p}+ O_R\biggl(\delta\sum_{\substack{p<q \\ p\not{\equiv} 3\, (\operatorname{mod}4)}}\frac{\log p}{p}+1\biggr)\leqslant \frac{1}{2}\log\log x+B, \end{aligned} \end{equation*} \notag $$
where $B=B(R)>0$. Thus,
$$ \begin{equation*} S_{q,l}\ll_R \frac{x^{-\delta/60}}{q(l-1)!}\biggl(\frac{1}{2}\log\log x+B\biggr)^{l-1} \end{equation*} \notag $$
and
$$ \begin{equation} \begin{aligned} \, N_3^{(2)} &\ll_R x\sum_{\substack{e^{2C}<q\leqslant x^{1/30} \\ q\not\equiv 3\, (\operatorname{mod}4)}}\frac{x^{-\delta/60}}{q\log q}\sum_{k-\eta\leqslant l\leqslant k-1}\frac{2^l(1/2\log\log x+B)^{l-1}}{(l-1)!}\notag \\ &\ll \frac{x(\log\log x+2B)^{k-1}}{(k-1)!}\sum_{q\leqslant x^{1/30}}\frac{x^{-\delta/60}}{q\log q}(1+R+\dots +R^{[\eta]}). \end{aligned} \end{equation} \tag{3.12} $$
Using the inequality
$$ \begin{equation*} 1+R+\dots+R^{[\eta]}\leqslant (R+2)^{\eta+1} \end{equation*} \notag $$
and the fact that
$$ \begin{equation*} (R+2)^\eta x^{-\delta/60}=x^{-1/\log q} \end{equation*} \notag $$
(which follows from the definitions (3.8) and (3.10) of $\eta$ and $\delta$, respectively), we see that the sum over primes $q$ in (3.12) is at most
$$ \begin{equation*} \begin{aligned} \, &(R+2) \sum_{q\leqslant x^{1/30}}\frac{(R+2)^{\eta}x^{-\delta/60}}{q\log q} \leqslant (R+2)\sum_{q\leqslant x^{1/2}}\frac{x^{-1/\log q}}{q\log q} \\ &\qquad\leqslant \frac{R+2}{\log x}\sum_{j\geqslant 1}\sum_{q\in (x^{2^{-(j+1)}}, \,x^{-2^j}]}\frac1q \,2^{j+1}\exp(-2^j) \ll_R \frac{1}{\log x}\,. \end{aligned} \end{equation*} \notag $$
Substituting this into (3.12) and taking (3.11) into account we find that
$$ \begin{equation} N_3 \ll_R \frac{x(\log\log x+2B)^{k-1}}{(k-1)!\,\log x}. \end{equation} \tag{3.13} $$

Combining (3.4), (3.6), (3.7) and (3.13) we complete the proof.

§ 4. Proof of Theorem 1.1

Writing $p-1=ab$, where $P^+(a)\leqslant x^{1/40}$ and $P^-(b)>x^{1/40}$, we can rewrite the sum $\Phi(x)$ as

$$ \begin{equation*} \Phi(x)=\sum_{\substack{a \leqslant x \\ P^+(a) \leqslant x^{1/40}}} \frac{1}{\tau(a)} \sum_{\substack{b \leqslant (x-1)/a \\ P^-(b) > x^{{1}/{40}} \\ ab +1\text{ is prime}}} \frac{1}{\tau(b)}. \end{equation*} \notag $$
Let $b = p_1^{\alpha_1} p_2^{\alpha_2} \dotsb p_s^{\alpha_s}$ be the prime factorization of $b$. Then
$$ \begin{equation*} x^{ (\alpha_1 + \alpha_2 + \dots + \alpha_s)/40} < b \leqslant x, \end{equation*} \notag $$
so that $\alpha_1+\dots+\alpha_s < 40$ and
$$ \begin{equation*} \tau(b)=(\alpha_1+1) \dotsb (\alpha_s+1) \leqslant 2^{\alpha_1+\dots+\alpha_s} < 2^{40}. \end{equation*} \notag $$
Therefore,
$$ \begin{equation*} \Phi(x) \geqslant 2^{-40} \sum_{\substack{a \leqslant x \\ P^+(a) \leqslant x^{1/40}}} \frac{1}{\tau(a)} \sum_{\substack{b \leqslant (x-1)/a \\ P^-(b) > x^{1/40} \\ ab +1\text{ is prime}}} 1 \geqslant 2^{-40} \sum_{\substack{a \leqslant x^{1/40} \\ a \text{ is even}}} \frac{1}{\tau(a)}F_a(x) . \end{equation*} \notag $$
Lemma 3.2 implies that
$$ \begin{equation*} \Phi(x) \geqslant \frac{c_2 \pi(x)}{\log x}\sum_{\substack{a \leqslant x^{1/40} \\ a \text{ is even}}} \frac{1}{\tau(a)\varphi(a)}-R_2, \end{equation*} \notag $$
where $c_2>0$ and
$$ \begin{equation*} 0\leqslant R_2 \leqslant \sum_{a\leqslant x^{1/40}}\sum_{d\leqslant x^{13/40}}|R(x;ad,1)| \leqslant \sum_{q\leqslant x^{7/20}}\tau(q)|R(x;q,1)|. \end{equation*} \notag $$
Using the trivial bound $|R(x;q,1)|\ll x/q$ and the Cauchy-Schwarz inequality we obtain
$$ \begin{equation*} \begin{aligned} \, R_2 &\ll x^{1/2} \sum_{q \leqslant x^{0.35}} \frac{\tau(q)}{q^{1/2}} (|R(x; q, 1)|)^{1/2} \\ &\leqslant x^{1/2} \biggl(\sum_{q \leqslant x^{0.35}} \frac{\tau^2(q)}{q}\biggr)^{1/2} \biggl(\sum_{q \leqslant x^{0.35}} |R(x; q, 1)|\biggr)^{1/2}. \end{aligned} \end{equation*} \notag $$
Applying the bound
$$ \begin{equation*} \sum_{n\leqslant x}\frac{\tau^2(n)}{n} \asymp (\log x)^4, \end{equation*} \notag $$
which follows from $\sum_{n\leqslant x}\tau^2(n)\asymp x(\log x)^3$ (see [13], Ch. III, Exercise 7) by partial summation, and the Bombieri-Vinogradov theorem we find that, for any fixed ${A>0}$,
$$ \begin{equation*} R_2 \ll_A x^{1/2}(\log x)^2 \frac{x^{1/2}}{(\log x)^{A/2}}= \frac{x}{(\log x)^{A/2-2}}. \end{equation*} \notag $$
Taking $A = 24$, say, we obtain
$$ \begin{equation*} R_2 \ll \frac{x}{(\log x)^{10}}. \end{equation*} \notag $$
Now we turn to the sum
$$ \begin{equation*} T_1=\sum_{\substack{a \leqslant x^{1/40} \\ a \text{ is even}}} \frac{1}{\tau(a) \varphi(a)}=\sum_{l \leqslant 0.5x^{1/40}} \frac{1}{\tau(2l)\varphi(2l)}. \end{equation*} \notag $$

Since $\tau(mn) \leqslant \tau(m)\tau(n)$ and $\varphi(n) \leqslant n$, we see that

$$ \begin{equation*} T_1\gg \sum_{l \leqslant 0.5x^{1/40}} \frac{1}{\tau(l)l}. \end{equation*} \notag $$
By (1.1), using partial summation we have
$$ \begin{equation*} T_1\gg (\log x)^{1/2}. \end{equation*} \notag $$
Hence
$$ \begin{equation*} \Phi(x)\gg \frac{\pi(x)}{(\log x)^{1/2}}-O\biggl( \frac{x}{(\log x)^{10}}\biggr)\gg \frac{x}{(\log x)^{3/2}}. \end{equation*} \notag $$

Theorem 1.1 is proved.

§ 5. Proof of Theorem 1.2

First we prove the lower bound. For any $z\geqslant2$,

$$ \begin{equation*} F(x)=\sum_{n \leqslant x} \frac{1}{\tau(n^2+1)}=\sum_{\substack{ab=n^2 + 1,\,n \leqslant x\\ P^+(a) \leqslant z,\, P^-(b) > z}} \frac{1}{\tau(ab)}. \end{equation*} \notag $$
Setting $z = x^{1/30}$ we have $\tau(b) \leqslant 2^{60}$, and now by Lemma 3.4,
$$ \begin{equation*} \begin{aligned} \, F(x) &\geqslant 2^{-60} \sum_{\substack{a \leqslant x^{1/30} \\ a \in \mathcal{M}}} \frac{1}{\tau(a)} W_a(x, x^{1/30})\gg \frac{x}{\log x} \sum_{\substack{a \leqslant x^{1/30} \\ a \in \mathcal{M}}} \frac{2^{\omega(a)}}{\tau(a)\varphi(a)} \\ &\geqslant \frac{x}{\log x}\sum_{\substack{a \leqslant x^{1/30} \\ a \in \mathcal{M} \\ a\text{ is square-free}}}\frac{1}{a}= \frac{x}{\log x}\sum_{\substack{a \leqslant x^{1/30} \\ a \in \mathcal{M}}} \frac{1}{a}\,\sum_{\delta^2\mid a}\mu(\delta). \end{aligned} \end{equation*} \notag $$
Changing the order of summation and using the fact that
$$ \begin{equation*} \sum_{n=1}^{\infty}\frac{1}{n^2}=\frac{\pi^2}{6}, \end{equation*} \notag $$
we have
$$ \begin{equation} F(x)\gg \frac{x}{\log x}\sum_{\substack{\delta \leqslant x^{1/60} \\ \delta \in \mathcal{M}}} \frac{\mu(\delta)}{\delta^2} \sum_{\substack{a \leqslant x^{1/30}\delta^{-2} \\ a \in \mathcal{M} }} \frac{1}{a} \geqslant \biggl( 2-\frac{\pi^2}{6}\biggr)\frac{x}{\log x}\sum_{\substack{a \leqslant x^{1/30} \\ a \in \mathcal{M}}} \frac{1}{a}. \end{equation} \tag{5.1} $$
It is well known (see [14], § 183) that for $y\geqslant 2$ one has
$$ \begin{equation*} \# \{a\leqslant y\colon p\mid a \ \Rightarrow\ p\equiv 1\ (\operatorname{mod} 4)\} \asymp \frac{y}{(\log y)^{1/2}}. \end{equation*} \notag $$

Using partial summation we obtain

$$ \begin{equation*} \sum_{\substack{a\leqslant y \\ p\mid a \ \Rightarrow\ p\equiv 1\, (\operatorname{mod} 4) }}\frac{1}{a}\asymp (\log y)^{1/2}. \end{equation*} \notag $$
Applying this to (5.1) we find that
$$ \begin{equation*} F(x)\gg \frac{x}{(\log x)^{1/2}}, \end{equation*} \notag $$
as required.

Now we prove the upper bound. By Lemma 3.5,

$$ \begin{equation*} \begin{aligned} \, F(x)&\leqslant \sum_{n\leqslant x}\frac{1}{2^{\omega(n^2+1)}} \\ &=\sum_{k\leqslant 2\log\log x}\frac{\#\{n\leqslant x\colon \omega(n^2+1)=k\}}{2^k} + O\biggl(\sum_{k>2\log\log x}\frac{x}{2^k}\biggr) \\ &\ll \frac{x}{\log x}\sum_{k=1}^{+\infty}\frac{((1/2)\log\log x+(1/2)B)^{k-1}}{(k-1)!} + O\biggl(\frac{x}{\log x}\biggr)\ll \frac{x}{(\log x)^{1/2}}. \end{aligned} \end{equation*} \notag $$

Theorem 1.2 is proved.


Bibliography

1. E. C. Titchmarsh, “A divisor problem”, Rend. Circ. Mat. Palermo, 54 (1930), 414–429  crossref  zmath
2. Yu. V. Linnik, “New versions and new uses of the dispersion method in binary additive problems”, Soviet Math. Dokl., 2 (1961), 468–471  mathnet  mathscinet  zmath
3. H. Halberstam, “Footnote to the Titchmarsh-Linnik divisor problem”, Proc. Amer. Math. Soc., 18 (1967), 187–188  crossref  mathscinet  zmath
4. E. Bombieri, J. B. Friedlander and H. Iwaniec, “Primes in arithmetic progressions to large moduli”, Acta Math., 156:3-4 (1986), 203–251  crossref  mathscinet  zmath
5. S. Ramanujan, “Some formulae in the analytic theory of numbers”, Messenger Math., 45 (1916), 81–84  mathscinet  zmath
6. V. V. Iudelevich, “On the Karatsuba divisor problem”, Izv. Math., 86:5 (2022), 992–1019  mathnet  crossref  mathscinet
7. P. Pollack, “Nonnegative multiplicative functions on sifted sets, and the square roots of $-1$ modulo shifted primes”, Glasg. Math. J., 62:1 (2020), 187–199  crossref  mathscinet  zmath
8. M. B. Barban and P. P. Vekhov, “Summation of multiplicative functions of polynomials”, Math. Notes, 5:6 (1969), 400–407  mathnet  crossref  mathscinet  zmath
9. K. Ford and H. Halberstam, “The Brun-Hooley sieve”, J. Number Theory, 81:2 (2000), 335–350  crossref  mathscinet  zmath
10. J. B. Rosser and L. Schoenfeld, “Approximate formulas for some functions of prime numbers”, Illinois J. Math., 6:1 (1962), 64–94  crossref  mathscinet  zmath
11. S. Uchiyama, “On some products involving primes”, Proc. Amer. Math. Soc., 28:2 (1971), 629–630  crossref  mathscinet  zmath
12. G. Tenenbaum, “Note sur les lois locales conjointes de la fonction nombre de facteurs premiers”, J. Number Theory, 188 (2018), 88–95  crossref  mathscinet  zmath
13. K. Prachar, Primzahlverteilung, Springer-Verlag, Berlin–Göttingen–Heidelberg, 1957, x+415 pp.  mathscinet  zmath
14. E. Landau, Handbuch der Lehre von der Verteilung der Primzahlen, 2 Bände, B. G. Teubner, Leipzig–Berlin, 1909, x+564 pp., ix+567–961 pp.  mathscinet  zmath

Citation: M. R. Gabdullin, S. V. Konyagin, V. V. Iudelevich, “Karatsuba's divisor problem and related questions”, Mat. Sb., 214:7 (2023), 27–41; Sb. Math., 214:7 (2023), 919–933
Citation in format AMSBIB
\Bibitem{GabKonIud23}
\by M.~R.~Gabdullin, S.~V.~Konyagin, V.~V.~Iudelevich
\paper Karatsuba's divisor problem and related questions
\jour Mat. Sb.
\yr 2023
\vol 214
\issue 7
\pages 27--41
\mathnet{http://mi.mathnet.ru/sm9815}
\crossref{https://doi.org/10.4213/sm9815}
\mathscinet{http://mathscinet.ams.org/mathscinet-getitem?mr=4681472}
\adsnasa{https://adsabs.harvard.edu/cgi-bin/bib_query?2023SbMat.214..919G}
\transl
\jour Sb. Math.
\yr 2023
\vol 214
\issue 7
\pages 919--933
\crossref{https://doi.org/10.4213/sm9815e}
\isi{https://gateway.webofknowledge.com/gateway/Gateway.cgi?GWVersion=2&SrcApp=Publons&SrcAuth=Publons_CEL&DestLinkType=FullRecord&DestApp=WOS_CPL&KeyUT=001146029300002}
\scopus{https://www.scopus.com/record/display.url?origin=inward&eid=2-s2.0-85180429388}
Linking options:
  • https://www.mathnet.ru/eng/sm9815
  • https://doi.org/10.4213/sm9815e
  • https://www.mathnet.ru/eng/sm/v214/i7/p27
  • Citing articles in Google Scholar: Russian citations, English citations
    Related articles in Google Scholar: Russian articles, English articles
    Математический сборник Sbornik: Mathematics
    Statistics & downloads:
    Abstract page:361
    Russian version PDF:14
    English version PDF:25
    Russian version HTML:153
    English version HTML:77
    References:22
    First page:13
     
      Contact us:
     Terms of Use  Registration to the website  Logotypes © Steklov Mathematical Institute RAS, 2024